Stack Exchange Network

Stack Exchange network consists of 183 Q&A communities including Stack Overflow , the largest, most trusted online community for developers to learn, share their knowledge, and build their careers.

Q&A for work

Connect and share knowledge within a single location that is structured and easy to search.

Null and Alternative hypothesis for multiple linear regression

I have 1 dependent variable and 3 independent variables.

I run multiple regression, and find that the p value for one of the independent variables is higher than 0.05 (95% is my confidence level).

I take that variable out and run it again. Both remaining independent variables have $p$-value less than 0.05 so I conclude I have my model.

Am I correct in thinking that initially, my null hypothesis is

$$H_0= β_1=β_2 = \dots =β_{k-1} = 0$$

and that the alternative hypothesis is

$$H_1=\textrm{At least one } β \neq 0 \textrm{ whilst } p<0.05$$

And that after the first regression, I do not reject, as one variable does not meet my confidence level needs...

So I run it again, and then reject the null as all $p$-values are significant?

Is what I have written accurate?

Edit: Thanks to Bob Jansen for improving this aesthetics of this post.

Harry's user avatar

2 Answers 2

The hypothesis $H_0: β_1=β_2=\dots =β_{k−1}=0$ is normally tested by the $F$-test for the regression.

You are carrying out 3 independent tests of your coefficients (Do you also have a constant in the regression or is the constant one of your three variables?) If you do three independent tests at a 5% level you have a probability of over 14% of finding one of the coefficients significant at the 5% level even if all coefficients are truly zero (the null hypothesis). This is often ignored but be careful. Even so, If the coefficient is close to significant I would think about the underlying theory before coming to a decision.

If you add dummies you will have a beta for each dummy

user1483's user avatar

  • $\begingroup$ Thanks for your response. I don't have a constant, all of my p-values are very significant (the least is a dummy variable at 0.039). What would my null hypothesis be? My knowledge is that I'm seeking p-values because that'd give me my model. I don't understand the technicalities of it and want to learn it :) $\endgroup$ –  Harry Commented Jan 7, 2015 at 22:36
  • $\begingroup$ I think you meant to say 14% of committing a type one error (probability of 0.14 of finding at least one of the coefficient significant when there true value is actually the null hypothesis value) $\endgroup$ –  Kamster Commented Jan 8, 2015 at 0:36
  • $\begingroup$ @Kamster Thanks. You are correct and I have amended my answer. $\endgroup$ –  user1483 Commented Jan 21, 2015 at 21:26

These are independent variables so the hypothesis applies to each parameter independently.

Andrew's user avatar

  • $\begingroup$ +1: Yes, you are right - but the rest of it should be fine $\endgroup$ –  vonjd Commented Jan 2, 2015 at 21:18
  • $\begingroup$ sorry, could you clarify? How do I change the equation so it applies to each parameter independently? And also, what is the effect of adding 3 dummy variables. Is it simply 2 more betas? Or do they require their own symbol $\endgroup$ –  Harry Commented Jan 4, 2015 at 0:32
  • $\begingroup$ It just means that you have an H_0 and an H_1 for every parameter. $\endgroup$ –  vonjd Commented Jan 4, 2015 at 11:33
  • $\begingroup$ Ok I see. Do you know the procedure for dummy variables? Are they just additional beta? Or is it more accurate to refer to them as delta? $\endgroup$ –  Harry Commented Jan 4, 2015 at 11:43
  • $\begingroup$ Maybe I have this wrong but isn't it true if you remain your individual significance levels at 0.05 that the probability of type one error (ie the probability that reject null hypothesis when it is actually true; significance level) will be greater than or equal 0.14 $\endgroup$ –  Kamster Commented Jan 8, 2015 at 0:43

Your Answer

Sign up or log in, post as a guest.

Required, but never shown

By clicking “Post Your Answer”, you agree to our terms of service and acknowledge you have read our privacy policy .

Not the answer you're looking for? Browse other questions tagged modelling or ask your own question .

  • Featured on Meta
  • Bringing clarity to status tag usage on meta sites
  • Announcing a change to the data-dump process

Hot Network Questions

  • Is there a way to resist spells or abilities with an AOE coming from my teammates, or exclude certain beings from the effect?
  • My supervisor wants me to switch to another software/programming language that I am not proficient in. What to do?
  • Stuck on Sokoban
  • 3D printed teffilin?
  • Parody of Fables About Authenticity
  • How much easier/harder would it be to colonize space if humans found a method of giving ourselves bodies that could survive in almost anything?
  • What does "seeing from one end of the world to the other" mean?
  • What's the average flight distance on a typical single engine aircraft?
  • How to count mismatches between two rows, column by column R?
  • Is there a phrase for someone who's really bad at cooking?
  • Is the ILIKE Operator in QGIS not working correctly?
  • Displaying a text in the center of the page
  • Rings demanding identity in the categorical context
  • Why was this lighting fixture smoking? What do I do about it?
  • Disable other-half popup when view splitting
  • Could someone tell me what this part of an A320 is called in English?
  • How to export a list of Datasets in separate sheets in XLSX?
  • Has the US said why electing judges is bad in Mexico but good in the US?
  • Does there always exist an algebraic formula for a function?
  • Why are complex coordinates outlawed in physics?
  • Is there a difference between these two layouts?
  • Is this screw inside a 2-prong receptacle a possible ground?
  • Do the amplitude and frequency of gravitational waves emitted by binary stars change as the stars get closer together?
  • Writing an i with a line over it instead of an i with a dot and a line over it

alternative hypothesis for multiple regression

Have a language expert improve your writing

Run a free plagiarism check in 10 minutes, generate accurate citations for free.

  • Knowledge Base

Multiple Linear Regression | A Quick Guide (Examples)

Published on February 20, 2020 by Rebecca Bevans . Revised on June 22, 2023.

Regression models are used to describe relationships between variables by fitting a line to the observed data. Regression allows you to estimate how a dependent variable changes as the independent variable(s) change.

Multiple linear regression is used to estimate the relationship between  two or more independent variables and one dependent variable . You can use multiple linear regression when you want to know:

  • How strong the relationship is between two or more independent variables and one dependent variable (e.g. how rainfall, temperature, and amount of fertilizer added affect crop growth).
  • The value of the dependent variable at a certain value of the independent variables (e.g. the expected yield of a crop at certain levels of rainfall, temperature, and fertilizer addition).

Table of contents

Assumptions of multiple linear regression, how to perform a multiple linear regression, interpreting the results, presenting the results, other interesting articles, frequently asked questions about multiple linear regression.

Multiple linear regression makes all of the same assumptions as simple linear regression :

Homogeneity of variance (homoscedasticity) : the size of the error in our prediction doesn’t change significantly across the values of the independent variable.

Independence of observations : the observations in the dataset were collected using statistically valid sampling methods , and there are no hidden relationships among variables.

In multiple linear regression, it is possible that some of the independent variables are actually correlated with one another, so it is important to check these before developing the regression model. If two independent variables are too highly correlated (r2 > ~0.6), then only one of them should be used in the regression model.

Normality : The data follows a normal distribution .

Linearity : the line of best fit through the data points is a straight line, rather than a curve or some sort of grouping factor.

Here's why students love Scribbr's proofreading services

Discover proofreading & editing

Multiple linear regression formula

The formula for a multiple linear regression is:

y = {\beta_0} + {\beta_1{X_1}} + … + {{\beta_n{X_n}} + {\epsilon}

  • … = do the same for however many independent variables you are testing

B_nX_n

To find the best-fit line for each independent variable, multiple linear regression calculates three things:

  • The regression coefficients that lead to the smallest overall model error.
  • The t statistic of the overall model.
  • The associated p value (how likely it is that the t statistic would have occurred by chance if the null hypothesis of no relationship between the independent and dependent variables was true).

It then calculates the t statistic and p value for each regression coefficient in the model.

Multiple linear regression in R

While it is possible to do multiple linear regression by hand, it is much more commonly done via statistical software. We are going to use R for our examples because it is free, powerful, and widely available. Download the sample dataset to try it yourself.

Dataset for multiple linear regression (.csv)

Load the heart.data dataset into your R environment and run the following code:

This code takes the data set heart.data and calculates the effect that the independent variables biking and smoking have on the dependent variable heart disease using the equation for the linear model: lm() .

Learn more by following the full step-by-step guide to linear regression in R .

To view the results of the model, you can use the summary() function:

This function takes the most important parameters from the linear model and puts them into a table that looks like this:

R multiple linear regression summary output

The summary first prints out the formula (‘Call’), then the model residuals (‘Residuals’). If the residuals are roughly centered around zero and with similar spread on either side, as these do ( median 0.03, and min and max around -2 and 2) then the model probably fits the assumption of heteroscedasticity.

Next are the regression coefficients of the model (‘Coefficients’). Row 1 of the coefficients table is labeled (Intercept) – this is the y-intercept of the regression equation. It’s helpful to know the estimated intercept in order to plug it into the regression equation and predict values of the dependent variable:

The most important things to note in this output table are the next two tables – the estimates for the independent variables.

The Estimate column is the estimated effect , also called the regression coefficient or r 2 value. The estimates in the table tell us that for every one percent increase in biking to work there is an associated 0.2 percent decrease in heart disease, and that for every one percent increase in smoking there is an associated .17 percent increase in heart disease.

The Std.error column displays the standard error of the estimate. This number shows how much variation there is around the estimates of the regression coefficient.

The t value column displays the test statistic . Unless otherwise specified, the test statistic used in linear regression is the t value from a two-sided t test . The larger the test statistic, the less likely it is that the results occurred by chance.

The Pr( > | t | ) column shows the p value . This shows how likely the calculated t value would have occurred by chance if the null hypothesis of no effect of the parameter were true.

Because these values are so low ( p < 0.001 in both cases), we can reject the null hypothesis and conclude that both biking to work and smoking both likely influence rates of heart disease.

When reporting your results, include the estimated effect (i.e. the regression coefficient), the standard error of the estimate, and the p value. You should also interpret your numbers to make it clear to your readers what the regression coefficient means.

Visualizing the results in a graph

It can also be helpful to include a graph with your results. Multiple linear regression is somewhat more complicated than simple linear regression, because there are more parameters than will fit on a two-dimensional plot.

However, there are ways to display your results that include the effects of multiple independent variables on the dependent variable, even though only one independent variable can actually be plotted on the x-axis.

Multiple regression in R graph

Here, we have calculated the predicted values of the dependent variable (heart disease) across the full range of observed values for the percentage of people biking to work.

To include the effect of smoking on the independent variable, we calculated these predicted values while holding smoking constant at the minimum, mean , and maximum observed rates of smoking.

Receive feedback on language, structure, and formatting

Professional editors proofread and edit your paper by focusing on:

  • Academic style
  • Vague sentences
  • Style consistency

See an example

alternative hypothesis for multiple regression

If you want to know more about statistics , methodology , or research bias , make sure to check out some of our other articles with explanations and examples.

  • Chi square test of independence
  • Statistical power
  • Descriptive statistics
  • Degrees of freedom
  • Pearson correlation
  • Null hypothesis

Methodology

  • Double-blind study
  • Case-control study
  • Research ethics
  • Data collection
  • Hypothesis testing
  • Structured interviews

Research bias

  • Hawthorne effect
  • Unconscious bias
  • Recall bias
  • Halo effect
  • Self-serving bias
  • Information bias

A regression model is a statistical model that estimates the relationship between one dependent variable and one or more independent variables using a line (or a plane in the case of two or more independent variables).

A regression model can be used when the dependent variable is quantitative, except in the case of logistic regression, where the dependent variable is binary.

Multiple linear regression is a regression model that estimates the relationship between a quantitative dependent variable and two or more independent variables using a straight line.

Linear regression most often uses mean-square error (MSE) to calculate the error of the model. MSE is calculated by:

  • measuring the distance of the observed y-values from the predicted y-values at each value of x;
  • squaring each of these distances;
  • calculating the mean of each of the squared distances.

Linear regression fits a line to the data by finding the regression coefficient that results in the smallest MSE.

Cite this Scribbr article

If you want to cite this source, you can copy and paste the citation or click the “Cite this Scribbr article” button to automatically add the citation to our free Citation Generator.

Bevans, R. (2023, June 22). Multiple Linear Regression | A Quick Guide (Examples). Scribbr. Retrieved August 30, 2024, from https://www.scribbr.com/statistics/multiple-linear-regression/

Is this article helpful?

Rebecca Bevans

Rebecca Bevans

Other students also liked, simple linear regression | an easy introduction & examples, an introduction to t tests | definitions, formula and examples, types of variables in research & statistics | examples, what is your plagiarism score.

Stack Exchange Network

Stack Exchange network consists of 183 Q&A communities including Stack Overflow , the largest, most trusted online community for developers to learn, share their knowledge, and build their careers.

Q&A for work

Connect and share knowledge within a single location that is structured and easy to search.

Writing hypothesis for linear multiple regression models

I struggle writing hypothesis because I get very much confused by reference groups in the context of regression models.

For my example I'm using the mtcars dataset. The predictors are wt (weight), cyl (number of cylinders), and gear (number of gears), and the outcome variable is mpg (miles per gallon).

Say all your friends think you should buy a 6 cylinder car, but before you make up your mind you want to know how 6 cylinder cars perform miles-per-gallon-wise compared to 4 cylinder cars because you think there might be a difference.

Would this be a fair null hypothesis (since 4 cylinder cars is the reference group)?: There is no difference between 6 cylinder car miles-per-gallon performance and 4 cylinder car miles-per-gallon performance.

Would this be a fair model interpretation ?: 6 cylinder vehicles travel fewer miles per gallon (p=0.010, β -4.00, CI -6.95 - -1.04) as compared to 4 cylinder vehicles when adjusting for all other predictors, thus rejecting the null hypothesis.

Sorry for troubling, and thanks in advance for any feedback!

enter image description here

  • multiple-regression
  • linear-model
  • interpretation

LuizZ's user avatar

Yes, you already got the right answer to both of your questions.

  • Your null hypothesis in completely fair. You did it the right way. When you have a factor variable as predictor, you omit one of the levels as a reference category (the default is usually the first one, but you also can change that). Then all your other levels’ coefficients are tested for a significant difference compared to the omitted category. Just like you did.

If you would like to compare 6-cylinder cars with 8-cylinder car, then you would have to change the reference category. In your hypothesis you just could had added at the end (or as a footnote): "when adjusting for weight and gear", but it is fine the way you did it.

  • Your model interpretation is correct : It is perfect the way you did it. You could even had said: "the best estimate is that 6 cylinder vehicles travel 4 miles per gallon less than 4 cylinder vehicles (p-value: 0.010; CI: -6.95, -1.04), when adjusting for weight and gear, thus rejecting the null hypothesis".

Let's assume that your hypothesis was related to gears, and you were comparing 4-gear vehicles with 3-gear vehicles. Then your result would be β: 0.65; p-value: 0.67; CI: -2.5, 3.8. You would say that: "There is no statistically significant difference between three and four gear cars in fuel consumption, when adjusting for weight and engine power, thus failing to reject the null hypothesis".

Your Answer

Sign up or log in, post as a guest.

Required, but never shown

By clicking “Post Your Answer”, you agree to our terms of service and acknowledge you have read our privacy policy .

Not the answer you're looking for? Browse other questions tagged r regression multiple-regression linear-model interpretation or ask your own question .

  • Featured on Meta
  • Announcing a change to the data-dump process
  • Bringing clarity to status tag usage on meta sites

Hot Network Questions

  • Is the ILIKE Operator in QGIS not working correctly?
  • How do we reconcile the story of the woman caught in adultery in John 8 and the man stoned for picking up sticks on Sabbath in Numbers 15?
  • Cannot open and HTML file stored on RAM-disk with a browser
  • How to disable Google Lens in search when using Google Chrome?
  • How did Oswald Mosley escape treason charges?
  • Who was the "Dutch author", "Bumstone Bumstone"?
  • Is it possible to accurately describe something without describing the rest of the universe?
  • TikZ -- Best strategy to choose points for the Hobby algorithm
  • All stationary martingales are constant?
  • What prevents a browser from saving and tracking passwords entered to a site?
  • Why does Jesus give an action of Yahweh as an example of evil?
  • Is this screw inside a 2-prong receptacle a possible ground?
  • Memory-optimizing arduino code to be able to print all files from SD card
  • Why is the movie titled "Sweet Smell of Success"?
  • Ring Buffer Implementation in C++
  • Does Vexing Bauble counter taxed 0 mana spells?
  • Whence “uniform distribution”?
  • I overstayed 90 days in Switzerland. I have EU residency and never got any stamps in passport. Can I exit/enter at airport without trouble?
  • Where does the energy in ion propulsion come from?
  • Is there a difference between these two layouts?
  • Does the order of ingredients while cooking matter to an extent that it changes the overall taste of the food?
  • What unique phenomena would be observed in a system around a hypervelocity star?
  • Dutch Public Transportation Electronic Payment Options
  • Encode a VarInt

alternative hypothesis for multiple regression

Null Hypothesis for Multiple Regression

Table of Contents

What is a Null Hypothesis and Why Does it Matter?

In multiple regression analysis, a null hypothesis is a crucial concept that plays a central role in statistical inference and hypothesis testing. A null hypothesis, denoted by H0, is a statement that proposes no significant relationship between the independent variables and the dependent variable. In other words, the null hypothesis suggests that the independent variables do not explain the variation in the dependent variable.

The null hypothesis is essential in multiple regression because it provides a basis for testing the significance of the regression coefficients. By formulating a null hypothesis, researchers can determine whether the observed relationships between variables are due to chance or if they reflect a real phenomenon. A well-crafted null hypothesis also helps to avoid false positives, ensuring that the results are not merely a result of chance.

In the context of multiple regression, the null hypothesis is typically tested against an alternative hypothesis, denoted by H1. The alternative hypothesis proposes that there is a significant relationship between the independent variables and the dependent variable. By comparing the null and alternative hypotheses , researchers can determine the probability of observing the results assuming that the null hypothesis is true. This probability, known as the p-value, is a critical component of hypothesis testing in multiple regression.

Formulating a null hypothesis for multiple regression is a critical step in the research process, as it directly impacts the interpretation of the results. A null hypothesis that is poorly formulated or irrelevant to the research question can lead to misleading conclusions and incorrect decisions. Therefore, it is essential to understand the role of the null hypothesis in multiple regression analysis and how to formulate it correctly.

https://www.youtube.com/watch?v=cpL38ZeIecE

How to Formulate a Null Hypothesis for Multiple Regression

Formulating a null hypothesis for multiple regression is a crucial step in the research process . A well-crafted null hypothesis provides a clear direction for the research and ensures that the results are meaningful and relevant. In this section, we will provide a step-by-step guide on how to formulate a null hypothesis for multiple regression.

Step 1: Identify the Research Question

The first step in formulating a null hypothesis is to identify the research question. The research question should be specific, clear, and concise, and it should guide the entire research process. For example, “Is there a significant relationship between the amount of exercise and blood pressure in adults?”

Step 2: Select the Dependent and Independent Variables

The next step is to select the dependent and independent variables . The dependent variable is the outcome variable that we are trying to predict, while the independent variables are the predictor variables that we use to explain the variation in the dependent variable. In our example, the dependent variable is blood pressure, and the independent variable is the amount of exercise.

Step 3: State the Null Hypothesis

The null hypothesis is a statement that proposes no significant relationship between the independent variables and the dependent variable. In our example, the null hypothesis would be “There is no significant relationship between the amount of exercise and blood pressure in adults.” This null hypothesis is denoted by H0.

Step 4: State the Alternative Hypothesis

The alternative hypothesis is a statement that proposes a significant relationship between the independent variables and the dependent variable. In our example, the alternative hypothesis would be “There is a significant relationship between the amount of exercise and blood pressure in adults.” This alternative hypothesis is denoted by H1.

By following these steps, researchers can formulate a clear and concise null hypothesis for multiple regression. A well-crafted null hypothesis provides a clear direction for the research and ensures that the results are meaningful and relevant. In the next section, we will discuss the importance of the null hypothesis in multiple regression modeling.

alternative hypothesis for multiple regression

The Role of Null Hypothesis in Multiple Regression Modeling

In multiple regression modeling, the null hypothesis plays a crucial role in guiding the analysis and interpretation of results. The null hypothesis serves as a benchmark against which the alternative hypothesis is tested, and its formulation has a direct impact on the outcome of the analysis.

The null hypothesis influences model interpretation by determining the significance of the regression coefficients. If the null hypothesis is rejected, it implies that the in dependent variable s have a significant effect on the dependent variable, and the regression coefficients can be used to make predictions. On the other hand , if the null hypothesis is not rejected, it suggests that the independent variables do not have a significant effect on the dependent variable, and the regression coefficients are not reliable.

The null hypothesis also affects coefficient estimation in multiple regression. The null hypothesis is used to test the significance of each regression coefficient, and if the null hypothesis is rejected, the coefficient is considered statistically significant. This, in turn, affects the interpretation of the results, as statistically significant coefficients are used to make predictions and draw conclusions.

Furthermore, the null hypothesis is essential for p-value calculation in multiple regression. The p-value represents the probability of observing the results assuming that the null hypothesis is true. A low p-value indicates that the null hypothesis can be rejected, implying that the independent variables have a significant effect on the dependent variable. A high p-value, on the other hand, suggests that the null hypothesis cannot be rejected, and the independent variables do not have a significant effect on the dependent variable.

In summary, the null hypothesis is a critical component of multiple regression modeling, as it guides the analysis and interpretation of results. Its formulation has a direct impact on model interpretation, coefficient estimation, and p-value calculation. By understanding the role of the null hypothesis in multiple regression, researchers can ensure that their analysis is accurate and reliable, leading to meaningful conclusions and informed decision-making.

alternative hypothesis for multiple regression

Understanding Type I and Type II Errors in Multiple Regression

In multiple regression analysis, Type I and Type II errors are critical concepts that researchers must understand to ensure accurate and reliable results. These errors occur when testing the null hypothesis, and their consequences can be far-reaching.

A Type I error occurs when the null hypothesis is rejected, but it is actually true. This means that the researcher has incorrectly concluded that there is a significant relationship between the in dependent variable s and the dependent variable. The probability of committing a Type I error is denoted by α (alpha) and is typically set to 0.05. A Type I error can lead to false conclusions and misinformed decision-making.

On the other hand , a Type II error occurs when the null hypothesis is not rejected, but it is actually false. This means that the researcher has failed to detect a significant relationship between the independent variables and the dependent variable. The probability of committing a Type II error is denoted by β (beta) and is related to the power of the test. A Type II error can lead to missed opportunities and incorrect assumptions.

The consequences of committing Type I and Type II errors can be significant. A Type I error can lead to the implementation of ineffective solutions or the allocation of resources to non-essential areas. A Type II error can lead to the failure to identify important relationships or the underestimation of the impact of independent variables.

To minimize the risk of Type I and Type II errors, researchers must carefully formulate the null hypothesis, select an appropriate significance level, and ensure adequate sample size and data quality. By understanding the concepts of Type I and Type II errors, researchers can ensure that their multiple regression analysis is accurate, reliable, and informative.

alternative hypothesis for multiple regression

Interpreting the Results of Multiple Regression Analysis

Once the multiple regression analysis is complete, interpreting the results is crucial to understanding the relationships between the independent variables and the dependent variable. In this section, we will discuss how to interpret the coefficient of determination (R-squared), F-statistic, and p-values.

The coefficient of determination, denoted by R-squared, measures the proportion of variance in the dependent variable that is explained by the independent variables. An R-squared value close to 1 indicates a strong relationship between the independent variables and the dependent variable, while a value close to 0 indicates a weak relationship . In multiple regression analysis, R-squared is used to evaluate the goodness of fit of the model.

The F-statistic is a measure of the overall significance of the regression model. It is used to test the null hypothesis that all the regression coefficients are equal to zero. A high F-statistic value indicates that the regression model is significant, and the independent variables have a significant effect on the dependent variable.

P-values are used to determine the significance of each regression coefficient. A p-value less than the significance level (typically 0.05) indicates that the regression coefficient is statistically significant, and the independent variable has a significant effect on the dependent variable. On the other hand, a p-value greater than the significance level indicates that the regression coefficient is not statistically significant, and the independent variable does not have a significant effect on the dependent variable.

When interpreting the results of multiple regression analysis, it is essential to consider the null hypothesis for multiple regression. The null hypothesis is used to test the significance of the regression coefficients, and its formulation has a direct impact on the interpretation of the results. By understanding the null hypothesis and its role in multiple regression analysis, researchers can ensure that their results are accurate and reliable.

Common Pitfalls to Avoid When Working with Null Hypotheses

When working with null hypotheses in multiple regression analysis, it is essential to avoid common pitfalls that can lead to inaccurate or misleading results. In this section, we will discuss some of the most common mistakes to avoid when working with null hypotheses.

One of the most critical mistakes is incorrect hypothesis formulation. A poorly formulated null hypothesis can lead to incorrect conclusions and misinformed decision-making. To avoid this, researchers must carefully identify the research question, select the dependent and independent variables , and state the null hypothesis clearly and concisely.

Inadequate sample size is another common pitfall. A sample size that is too small can lead to inaccurate estimates of the regression coefficients and p-values, making it difficult to draw meaningful conclusions. Researchers must ensure that the sample size is sufficient to detect significant relationships between the independent variables and the dependent variable.

Misinterpretation of results is also a common mistake. Researchers must be careful not to overinterpret the results of multiple regression analysis, especially when it comes to the null hypothesis. A failure to reject the null hypothesis does not necessarily mean that there is no significant relationship between the independent variables and the dependent variable. Rather, it may indicate that the sample size is too small or the data is too noisy to detect a significant relationship.

Additionally, researchers must avoid ignoring the assumptions of multiple regression analysis. Violating the assumptions of linearity, independence, homoscedasticity, normality, and no or little multicollinearity can lead to inaccurate results and incorrect conclusions. By checking the assumptions of multiple regression analysis, researchers can ensure that the results are reliable and accurate.

Finally, researchers must avoid using multiple regression analysis as a black box . Multiple regression analysis is a powerful tool, but it requires a deep understanding of the underlying statistical concepts and assumptions. By understanding the null hypothesis for multiple regression and its role in statistical inference and hypothesis testing, researchers can ensure that their results are accurate, reliable, and informative.

alternative hypothesis for multiple regression

Real-World Applications of Multiple Regression Analysis

Multiple regression analysis has numerous real-world applications across various fields, including finance, marketing, healthcare, and more. In this section, we will explore some of the most significant applications of multiple regression analysis.

In finance, multiple regression analysis is used to predict stock prices, analyze portfolio risk, and identify factors that influence investment returns. For instance, a financial analyst may use multiple regression to examine the relationship between a company’s stock price and various economic indicators, such as GDP, inflation rate, and unemployment rate.

In marketing, multiple regression analysis is employed to analyze customer behavior, predict sales, and optimize marketing campaigns. Marketers may use multiple regression to identify the factors that influence customer purchasing decisions, such as demographics, advertising spend, and price.

In healthcare, multiple regression analysis is used to identify risk factors for diseases, predict patient outcomes, and evaluate the effectiveness of treatments. For example, a healthcare researcher may use multiple regression to examine the relationship between patient characteristics, such as age, gender, and lifestyle, and the risk of developing a particular disease.

In addition to these fields, multiple regression analysis has applications in economics, social sciences, and environmental studies. It is a powerful tool for analyzing complex relationships between variables and making informed decisions.

In all these applications, the null hypothesis for multiple regression plays a critical role in statistical inference and hypothesis testing. By formulating a clear and concise null hypothesis, researchers can ensure that their results are accurate, reliable, and informative.

By understanding the real-world applications of multiple regression analysis, researchers and practitioners can unlock the full potential of this powerful statistical technique and make data-driven decisions that drive business success and improve lives.

alternative hypothesis for multiple regression

Best Practices for Implementing Multiple Regression in Your Research

When implementing multiple regression in research, it is essential to follow best practices to ensure accurate, reliable, and informative results. In this section, we will discuss some of the most critical best practices for implementing multiple regression in research.

Data Preparation: Before conducting multiple regression analysis, it is crucial to prepare the data properly. This includes checking for missing values, outliers, and multicollinearity, as well as transforming variables to meet the assumptions of multiple regression.

Model Validation: Validating the multiple regression model is critical to ensuring that the results are accurate and reliable. This includes checking the model’s assumptions, such as linearity, independence, homoscedasticity, normality, and no or little multicollinearity.

Result Reporting: When reporting the results of multiple regression analysis, it is essential to provide clear and concise information about the model, including the null hypothesis for multiple regression, the coefficient of determination (R-squared), F-statistic, and p-values.

Interpretation of Results: Interpreting the results of multiple regression analysis requires a deep understanding of the null hypothesis for multiple regression and its role in statistical inference and hypothesis testing. Researchers must be careful not to overinterpret the results, especially when it comes to the null hypothesis.

Avoiding Common Pitfalls: Finally, researchers must avoid common pitfalls when working with null hypotheses in multiple regression, such as incorrect hypothesis formulation, inadequate sample size, and misinterpretation of results.

By following these best practices, researchers can ensure that their multiple regression analysis is accurate, reliable, and informative, and that the results are useful for making informed decisions.

Remember, the null hypothesis for multiple regression is a critical component of statistical inference and hypothesis testing, and it plays a vital role in ensuring that the results of multiple regression analysis are accurate and reliable.

alternative hypothesis for multiple regression

Advanced Statistics using R

Applied Data Science Meeting, July 4-6, 2023, Shanghai, China . Register for the workshops: (1) Deep Learning Using R, (2) Introduction to Social Network Analysis, (3) From Latent Class Model to Latent Transition Model Using Mplus, (4) Longitudinal Data Analysis, and (5) Practical Mediation Analysis. Click here for more information .

  • Example Datasets
  • Basics of R
  • Graphs in R

Hypothesis testing

  • Confidence interval
  • Simple Regression
  • Multiple Regression
  • Logistic regression
  • Moderation analysis
  • Mediation analysis
  • Path analysis
  • Factor analysis
  • Multilevel regression
  • Longitudinal data analysis
  • Power analysis

Multiple Linear Regression

The general purpose of multiple regression (the term was first used by Pearson, 1908), as a generalization of simple linear regression, is to learn about how several independent variables or predictors (IVs) together predict a dependent variable (DV). Multiple regression analysis often focuses on understanding (1) how much variance in a DV a set of IVs explain and (2) the relative predictive importance of IVs in predicting a DV.

In the social and natural sciences, multiple regression analysis is very widely used in research. Multiple regression allows a researcher to ask (and hopefully answer) the general question "what is the best predictor of ...". For example, educational researchers might want to learn what the best predictors of success in college are. Psychologists may want to determine which personality dimensions best predicts social adjustment.

Multiple regression model

A general multiple linear regression model at the population level can be written as

\[y_{i}=\beta_{0}+\beta_{1}x_{1i}+\beta_{2}x_{2i}+\ldots+\beta_{k}x_{ki}+\varepsilon_{i} \]

  • $y_{i}$: the observed score of individual $i$ on the DV.
  • $x_{1},x_{2},\ldots,x_{k}$ : a set of predictors.
  • $x_{1i}$: the observed score of individual $i$ on IV 1; $x_{ki}$: observed score of individual $i$ on IV $k$.
  • $\beta_{0}$: the intercept at the population level, representing the predicted $y$ score when all the independent variables have their values at 0.
  • $\beta_{1},\ldots,\beta_{k}$: regression coefficients at the population level; $\beta_{1}$: representing the amount predicted $y$ changes when $x_{1}$ changes in 1 unit while holding the other IVs constant; $\beta_{k}$: representing the amount predicted $y$ changes when $x_{k}$ changes in 1 unit while holding the other IVs constant.
  • $\varepsilon$: unobserved errors with mean 0 and variance $\sigma^{2}$.

Parameter estimation

The least squares method used for the simple linear regression analysis can also be used to estimate the parameters in a multiple regression model. The basic idea is to minimize the sum of squared residuals or errors. Let $b_{0},b_{1},\ldots,b_{k}$ represent the estimated regression coefficients.The individual $i$'s residual $e_{i}$ is the difference between the observed $y_{i}$ and the predicted $y_{i}$

\[ e_{i}=y_{i}-\hat{y}_{i}=y_{i}-b_{0}-b_{1}x_{1i}-\ldots-b_{k}x_{ki}.\]

The sum of squared residuals is

\[ SSE=\sum_{i=1}^{n}e_{i}^{2}=\sum_{i=1}^{n}(y_{i}-\hat{y}_{i})^{2}. \]

By minimizing $SSE$, the regression coefficient estimates can be obtained as

\[ \boldsymbol{b}=(\boldsymbol{X}'\boldsymbol{X})^{-1}\boldsymbol{X}'\boldsymbol{y}=(\sum\boldsymbol{x}_{i}\boldsymbol{x}_{i}')^{-1}(\sum\boldsymbol{x}_{i}\boldsymbol{y}_{i}). \]

How well the multiple regression model fits the data can be assessed using the $R^{2}$. Its calculation is the same as for the simple regression

\[\begin{align*} R^{2} & = & 1-\frac{\sum e_{i}^{2}}{\sum_{i=1}^{n}(y_{i}-\bar{y})^{2}}\\& = & \frac{\text{Variation explained by IVs}}{\text{Total variation}} \end{align*}. \]

In multiple regression, $R^{2}$ is the total proportion of variation in $y$ explained by the multiple predictors.

The $R^{2}$ increases or at least is the same with the inclusion of more predictors. However, with more predators, the model becomes more complex and potentially more difficult to interpret. In order to take into consideration of the model complexity, the adjusted $R^{2}$ has been defined, which is calculated as

\[aR^{2}=1-(1-R^{2})\frac{n-1}{n-k-1}.\]

Hypothesis testing of regression coefficient(s)

With the estimates of regression coefficients and their standard errors estimates, we can conduct hypothesis testing for one, a subset, or all regression coefficients.

Testing a single regression coefficient

At first, we can test the significance of the coefficient for a single predictor. In this situation, the null and alternative hypotheses are

\[ H_{0}:\beta_{j}=0\text{ vs }H_{1}:\beta_{j}\neq0 \]

with $\beta_{j}$ denoting the regression coefficient of $x_{j}$ at the population level.

As in the simple regression, we use a test statistic

\[ t_{j}=\frac{b_{j} - \beta{j} }{s.e.(b_{j})}\]

where $b_{j}$ is the estimated regression coefficient of $x_{j}$ using data from a sample. If the null hypothesis is true and $\beta_j = 0$, the test statistic follows a t-distribution with degrees of freedom \(n-k-1\) where \(k\) is the number of predictors.

One can also test the significance of \(\beta_j\) by constructing a confidence interval for it. Based on a t distribution, the \(100(1-\alpha)%\) confidence interval is

\[ [b_{j}+t_{n-k-1}(\alpha/2)*s.e.(b_{j}),\;b_{j}+t_{n-k-1}(1-\alpha/2)*s.e.(b_{j})]\]

where $t_{n-k-1}(\alpha/2)$ is the $\alpha/2$ percentile of the t distribution. As previously discussed, if the confidence interval includes 0, the regression coefficient is not statistically significant at the significance level $\alpha$.

Testing all the regression coefficients together (overall model fit)

Given the multiple predictors, we can also test whether all of the regression coefficients are 0 at the same time. This is equivalent to test whether all predictors combined can explained a significant portion of the variance of the outcome variable. Since $R^2$ is a measure of the variance explained, this test is naturally related to it.

For this hypothesis testing, the null and alternative hypothesis are

\[H_{0}:\beta_{1}=\beta_{2}=\ldots=\beta_{k}=0\]

\[H_{1}:\text{ at least one of the regression coefficients is different from 0}.\]

In this kind of test, an F test is used. The F-statistic is defined as

\[F=\frac{n-k-1}{k}\frac{R^{2}}{1-R^{2}}.\]

It follows an F-distribution with degrees of freedom $k$ and $n-k-1$ when the null hypothesis is true. Given an F statistic, its corresponding p-value can be calculated from the F distribution as shown below. Note that we only look at one side of the distribution because the extreme values should be on the large value side.

Testing a subset of the regression coefficients

We can also test whether a subset of $p$ regression coefficients, e.g., $p$ from 1 to the total number coefficients $k$, are equal to zero. For convenience, we can rearrange all the $p$ regression coefficients to be the first $p$ coefficients. Therefore, the null hypothesis should be

\[H_{0}:\beta_{1}=\beta_{2}=\ldots=\beta_{p}=0\]

and the alternative hypothesis is that at least one of them is not equal to 0.

As for testing the overall model fit, an F test can be used here. In this situation, the F statistic can be calculated as

\[F=\frac{n-k-1}{p}\frac{R^{2}-R_{0}^{2}}{1-R^{2}},\]

which follows an F-distribution with degrees of freedom $p$ and $n-k-1$. $R^2$ is for the regression model with all the predictors and $R_0^2$ is from the regression model without the first $p$ predictors $x_{1},x_{2},\ldots,x_{p}$ but with the rest predictors $x_{p+1},x_{p+2},\ldots,x_{k}$.

Intuitively, this test determine whether the variance explained by the first \(p\) predictors above and beyond the $k-p$ predictors is significance or not. That is also the increase in R-squared.

As an example, suppose that we wanted to predict student success in college. Why might we want to do this? There's an ongoing debate in college and university admission offices (and in the courts) regarding what factors should be considered important in deciding which applicants to admit. Should admissions officers pay most attention to more easily quantifiable measures such as high school GPA and SAT scores? Or should they give more weight to more subjective measures such as the quality of letters of recommendation? What are the pros and cons of the approaches? Of course, how we define college success is also an open question. For the sake of this example, let's measure college success using college GPA.

In this example, we use a set of simulated data (generated by us). The data are saved in the file gpa.csv. As shown below, the sample size is 100 and there are 4 variables: college GPA (c.gpa), high school GPA (h.gpa), SAT, and quality of recommendation letters (recommd).

Graph the data

Before fitting a regression model, we should check the relationship between college GPA and each predictor through a scatterplot. A scatterplot can tell us the form of relationship, e.g., linear, nonlinear, or no relationship, the direction of relationship, e.g., positive or negative, and the strength of relationship, e.g., strong, moderate, or weak. It can also identify potential outliers.

The scatterplots between college GPA and the three potential predictors are given below. From the plots, we can roughly see all three predictors are positively related to the college GPA. The relationship is close to linear and the relationship seems to be stronger for high school GPA and SAT than for the quality of recommendation letters.

Descriptive statistics

Next, we can calculate some summary statistics to explore our data further. For each variable, we calculate 6 numbers: minimum, 1st quartile, median, mean, 3rd quartile, and maximum. Those numbers can be obtained using the summary() function. To look at the relationship among the variables, we can calculate the correlation matrix using the correlation function cor() .

Based on the correlation matrix, the correlation between college GPA and high school GPA is about 0.545, which is larger than that (0.523) between college GPA and SAT, in turn larger than that (0.35) between college GPA and quality of recommendation letters.

Fit a multiple regression model

As for the simple linear regression, The multiple regression analysis can be carried out using the lm() function in R. From the output, we can write out the regression model as

\[ c.gpa = -0.153+ 0.376 \times h.gpa + 0.00122 \times SAT + 0.023 \times recommd \]

Interpret the results / output

From the output, we see the intercept is -0.153. Its immediate meaning is that when all predictors' values are 0, the predicted college GPA is -0.15. This clearly does not make much sense because one would never get a negative GPA, which results from the unrealistic presumption that the predictors can take the value of 0.

The regression coefficient for the predictor high school GPA (h.gpa) is 0.376. This can be interpreted as keeping SAT and recommd scores constant , the predicted college GPA would increase 0.376 with a unit increase in high school GPA.This is again might be problematic because it might be impossible to increase high school GPA while keeping the other two predictors unchanged. The other two regression coefficients can be interpreted in the same way.

From the output, we can also see that the multiple R-squared ($R^2$) is 0.3997. Therefore, about 40% of the variation in college GPA can be explained by the multiple linear regression with h.GPA, SAT, and recommd as the predictors. The adjusted $R^2$ is slightly smaller because of the consideration of the number of predictors. In fact,

\[ \begin{eqnarray*} aR^{2} & = & 1-(1-R^{2})\frac{n-1}{n-k-1}\\& = & 1-(1-.3997)\frac{100-1}{100-3-1}\\& = & .3809 \end{eqnarray*} \]

Testing Individual Regression Coefficient

For any regression coefficients for the three predictors (also the intercept), a t test can be conducted. For example, for high school GPA, the estimated coefficient is 0.376 with the standard error 0.114. Therefore, the corresponding t statistic is \(t = 0.376/0.114 = 3.294\). Since the statistic follows a t distribution with the degrees of freedom \(df = n - k - 1 = 100 - 3 -1 =96\), we can obtain the p-value as \(p = 2*(1-pt(3.294, 96))= 0.0013\). Since the p-value is less than 0.05, we conclude the coefficient is statistically significant. Note the t value and p-value are directly provided in the output.

Overall model fit (testing all coefficients together)

To test all coefficients together or the overall model fit, we use the F test. Given the $R^2$, the F statistic is

\[ \begin{eqnarray*} F & = & \frac{n-k-1}{k}\frac{R^{2}}{1-R^{2}}\\ & = & \left(\frac{100-3-1}{3}\right)\times \left(\frac{0.3997}{1-.3997}\right )=21.307\end{eqnarray*} \]

which follows the F distribution with degrees of freedom $df1=k=3$ and $df2=n-k-1=96$. The corresponding p-value is 1.160e-10. Note that this information is directly shown in the output as " F-statistic: 21.31 on 3 and 96 DF, p-value: 1.160e-10 ".

Therefore, at least one of the regression coefficients is statistically significantly different from 0. Overall, the three predictors explained a significant portion of the variance in college GPA. The regression model with the 3 predictors is significantly better than the regression model with intercept only (i.e., predict c.gpa by the mean of c.gpa).

Testing a subset of regression coefficients

Suppose we are interested in testing whether the regression coefficients of high school GPA and SAT together are significant or not. Alternative, we want to see above and beyond the quality of recommendation letters, whether the two predictors can explain a significant portion of variance in college GPA. To conduct the test, we need to fit two models:

  • A full model: which consists of all the predictors to predict c.gpa by intercept, h.gpa, SAT, and recommd.
  • A reduced model: obtained by removing the predictors to be tested in the full model.

From the full model, we can get the $R^2 = 0.3997$ with all three predictors and from the reduced model, we can get the $R_0^2 = 0.1226$ with only quality of recommendation letters. Then the F statistic is constructed as

\[F=\frac{n-k-1}{p}\frac{R^{2}-R_{0}^{2}}{1-R^{2}}=\left(\frac{100-3-1}{2}\right )\times\frac{.3997-.1226}{1-.3997}=22.157.\]

Using the F distribution with the degrees of freedom $p=2$ (the number of coefficients to be tested) and $n-k-1 = 96$, we can get the p-value close to 0 ($p=1.22e-08$).

Note that the test conducted here is based on the comparison of two models. In R, if there are two models, they can be compared conveniently using the R function anova() . As shown below, we obtain the same F statistic and p-value.

To cite the book, use: Zhang, Z. & Wang, L. (2017-2022). Advanced statistics using R . Granger, IN: ISDSA Press. https://doi.org/10.35566/advstats. ISBN: 978-1-946728-01-2. To take the full advantage of the book such as running analysis within your web browser, please subscribe .

User Preferences

Content preview.

Arcu felis bibendum ut tristique et egestas quis:

  • Ut enim ad minim veniam, quis nostrud exercitation ullamco laboris
  • Duis aute irure dolor in reprehenderit in voluptate
  • Excepteur sint occaecat cupidatat non proident

Keyboard Shortcuts

6.4 - the hypothesis tests for the slopes.

At the beginning of this lesson, we translated three different research questions pertaining to heart attacks in rabbits ( Cool Hearts dataset ) into three sets of hypotheses we can test using the general linear F -statistic. The research questions and their corresponding hypotheses are:

Hypotheses 1

Is the regression model containing at least one predictor useful in predicting the size of the infarct?

  • \(H_{0} \colon \beta_{1} = \beta_{2} = \beta_{3} = 0\)
  • \(H_{A} \colon\) At least one \(\beta_{j} ≠ 0\) (for j = 1, 2, 3)

Hypotheses 2

Is the size of the infarct significantly (linearly) related to the area of the region at risk?

  • \(H_{0} \colon \beta_{1} = 0 \)
  • \(H_{A} \colon \beta_{1} \ne 0 \)

Hypotheses 3

(Primary research question) Is the size of the infarct area significantly (linearly) related to the type of treatment upon controlling for the size of the region at risk for infarction?

  • \(H_{0} \colon \beta_{2} = \beta_{3} = 0\)
  • \(H_{A} \colon \) At least one \(\beta_{j} ≠ 0\) (for j = 2, 3)

Let's test each of the hypotheses now using the general linear F -statistic:

\(F^*=\left(\dfrac{SSE(R)-SSE(F)}{df_R-df_F}\right) \div \left(\dfrac{SSE(F)}{df_F}\right)\)

To calculate the F -statistic for each test, we first determine the error sum of squares for the reduced and full models — SSE ( R ) and SSE ( F ), respectively. The number of error degrees of freedom associated with the reduced and full models — \(df_{R}\) and \(df_{F}\), respectively — is the number of observations, n , minus the number of parameters, p , in the model. That is, in general, the number of error degrees of freedom is n - p . We use statistical software, such as Minitab's F -distribution probability calculator, to determine the P -value for each test.

Testing all slope parameters equal 0 Section  

To answer the research question: "Is the regression model containing at least one predictor useful in predicting the size of the infarct?" To do so, we test the hypotheses:

  • \(H_{0} \colon \beta_{1} = \beta_{2} = \beta_{3} = 0 \)
  • \(H_{A} \colon\) At least one \(\beta_{j} \ne 0 \) (for j = 1, 2, 3)

The full model

The full model is the largest possible model — that is, the model containing all of the possible predictors. In this case, the full model is:

\(y_i=(\beta_0+\beta_1x_{i1}+\beta_2x_{i2}+\beta_3x_{i3})+\epsilon_i\)

The error sum of squares for the full model, SSE ( F ), is just the usual error sum of squares, SSE , that appears in the analysis of variance table. Because there are 4 parameters in the full model, the number of error degrees of freedom associated with the full model is \(df_{F} = n - 4\).

The reduced model

The reduced model is the model that the null hypothesis describes. Because the null hypothesis sets each of the slope parameters in the full model equal to 0, the reduced model is:

\(y_i=\beta_0+\epsilon_i\)

The reduced model suggests that none of the variations in the response y is explained by any of the predictors. Therefore, the error sum of squares for the reduced model, SSE ( R ), is just the total sum of squares, SSTO , that appears in the analysis of variance table. Because there is only one parameter in the reduced model, the number of error degrees of freedom associated with the reduced model is \(df_{R} = n - 1 \).

Upon plugging in the above quantities, the general linear F -statistic:

\(F^*=\dfrac{SSE(R)-SSE(F)}{df_R-df_F} \div \dfrac{SSE(F)}{df_F}\)

becomes the usual " overall F -test ":

\(F^*=\dfrac{SSR}{3} \div \dfrac{SSE}{n-4}=\dfrac{MSR}{MSE}\)

That is, to test \(H_{0}\) : \(\beta_{1} = \beta_{2} = \beta_{3} = 0 \), we just use the overall F -test and P -value reported in the analysis of variance table:

Analysis of Variance

Source DF Adj SS Adj MS F- Value P-Value
Regression 3 0.95927 0.31976 16.43 0.000
Area 1 0.63742 0.63742 32.75 0.000
X2 1 0.29733 0.29733 15.28 0.001
X3 1 0.01981 0.01981 1.02 0.322
Error 28 0.54491 0.01946    
31 1.50418      

Regression Equation

Inf = - 0.135 + 0.613 Area - 0.2435 X2 - 0.0657 X3

There is sufficient evidence ( F = 16.43, P < 0.001) to conclude that at least one of the slope parameters is not equal to 0.

In general, to test that all of the slope parameters in a multiple linear regression model are 0, we use the overall F -test reported in the analysis of variance table.

Testing one slope parameter is 0 Section  

Now let's answer the second research question: "Is the size of the infarct significantly (linearly) related to the area of the region at risk?" To do so, we test the hypotheses:

Again, the full model is the model containing all of the possible predictors:

The error sum of squares for the full model, SSE ( F ), is just the usual error sum of squares, SSE . Alternatively, because the three predictors in the model are \(x_{1}\), \(x_{2}\), and \(x_{3}\), we can denote the error sum of squares as SSE (\(x_{1}\), \(x_{2}\), \(x_{3}\)). Again, because there are 4 parameters in the model, the number of error degrees of freedom associated with the full model is \(df_{F} = n - 4 \).

Because the null hypothesis sets the first slope parameter, \(\beta_{1}\), equal to 0, the reduced model is:

\(y_i=(\beta_0+\beta_2x_{i2}+\beta_3x_{i3})+\epsilon_i\)

Because the two predictors in the model are \(x_{2}\) and \(x_{3}\), we denote the error sum of squares as SSE (\(x_{2}\), \(x_{3}\)). Because there are 3 parameters in the model, the number of error degrees of freedom associated with the reduced model is \(df_{R} = n - 3\).

The general linear statistic:

simplifies to:

\(F^*=\dfrac{SSR(x_1|x_2, x_3)}{1}\div \dfrac{SSE(x_1,x_2, x_3)}{n-4}=\dfrac{MSR(x_1|x_2, x_3)}{MSE(x_1,x_2, x_3)}\)

Getting the numbers from the Minitab output:

we determine that the value of the F -statistic is:

\(F^* = \dfrac{SSR(x_1 \vert x_2, x_3)}{1} \div \dfrac{SSE(x_1, x_2, x_3)}{28} = \dfrac{0.63742}{0.01946}=32.7554\)

The P -value is the probability — if the null hypothesis were true — that we would get an F -statistic larger than 32.7554. Comparing our F -statistic to an F -distribution with 1 numerator degree of freedom and 28 denominator degrees of freedom, Minitab tells us that the probability is close to 1 that we would observe an F -statistic smaller than 32.7554:

F distribution with 1 DF in Numerator and 28 DF in denominator

x P ( X ≤x )
32.7554 1.00000

Therefore, the probability that we would get an F -statistic larger than 32.7554 is close to 0. That is, the P -value is < 0.001. There is sufficient evidence ( F = 32.8, P < 0.001) to conclude that the size of the infarct is significantly related to the size of the area at risk after the other predictors x2 and x3 have been taken into account.

But wait a second! Have you been wondering why we couldn't just use the slope's t -statistic to test that the slope parameter, \(\beta_{1}\), is 0? We can! Notice that the P -value ( P < 0.001) for the t -test ( t * = 5.72):

Coefficients

Term Coef SE Coef T-Value P-Value VIF
Constant -0.135 0.104 -1.29 0.206  
Area 0.613 0.107 5.72 0.000 1.14
X2 -0.2435 0.0623 -3.91 0.001 1.44
X3 -0.0657 0.0651 -1.01 0.322 1.57

is the same as the P -value we obtained for the F -test. This will always be the case when we test that only one slope parameter is 0. That's because of the well-known relationship between a t -statistic and an F -statistic that has one numerator degree of freedom:

\(t_{(n-p)}^{2}=F_{(1, n-p)}\)

For our example, the square of the t -statistic, 5.72, equals our F -statistic (within rounding error). That is:

\(t^{*2}=5.72^2=32.72=F^*\)

So what have we learned in all of this discussion about the equivalence of the F -test and the t -test? In short:

Compare the output obtained when \(x_{1}\) = Area is entered into the model last :

Term Coef SE Coef T-Value P-Value VIF
Constant -0.135 0.104 -1.29 0.206  
X2 -0.2435 0.0623 -3.91 0.001 1.44
X3 -0.0657 0.0651 -1.01 0.322 1.57
Area 0.613 0.107 5.72 0.000 1.14

Inf = - 0.135 - 0.2435 X2 - 0.0657 X3 + 0.613 Area

to the output obtained when \(x_{1}\) = Area is entered into the model first :

The t -statistic and P -value are the same regardless of the order in which \(x_{1}\) = Area is entered into the model. That's because — by its equivalence to the F -test — the t -test for one slope parameter adjusts for all of the other predictors included in the model.

  • We can use either the F -test or the t -test to test that only one slope parameter is 0. Because the t -test results can be read right off of the Minitab output, it makes sense that it would be the test that we'll use most often.
  • But, we have to be careful with our interpretations! The equivalence of the t -test to the F -test has taught us something new about the t -test. The t -test is a test for the marginal significance of the \(x_{1}\) predictor after the other predictors \(x_{2}\) and \(x_{3}\) have been taken into account. It does not test for the significance of the relationship between the response y and the predictor \(x_{1}\) alone.

Testing a subset of slope parameters is 0 Section  

Finally, let's answer the third — and primary — research question: "Is the size of the infarct area significantly (linearly) related to the type of treatment upon controlling for the size of the region at risk for infarction?" To do so, we test the hypotheses:

  • \(H_{0} \colon \beta_{2} = \beta_{3} = 0 \)
  • \(H_{A} \colon\) At least one \(\beta_{j} \ne 0 \) (for j = 2, 3)

Because the null hypothesis sets the second and third slope parameters, \(\beta_{2}\) and \(\beta_{3}\), equal to 0, the reduced model is:

\(y_i=(\beta_0+\beta_1x_{i1})+\epsilon_i\)

The ANOVA table for the reduced model is:

Source DF Adj SS Adj MS F- Value P-Value
Regression 1 0.6249 0.62492 21.32 0.000
Area 1 0.6249 0.62492 21.32 0.000
Error 30 0.8793 0.02931    
31 1.5042      

Because the only predictor in the model is \(x_{1}\), we denote the error sum of squares as SSE (\(x_{1}\)) = 0.8793. Because there are 2 parameters in the model, the number of error degrees of freedom associated with the reduced model is \(df_{R} = n - 2 = 32 – 2 = 30\).

\begin{align} F^*&=\dfrac{SSE(R)-SSE(F)}{df_R-df_F} \div\dfrac{SSE(F)}{df_F}\\&=\dfrac{0.8793-0.54491}{30-28} \div\dfrac{0.54491}{28}\\&= \dfrac{0.33439}{2} \div 0.01946\\&=8.59.\end{align}

Alternatively, we can calculate the F-statistic using a partial F-test :

\begin{align}F^*&=\dfrac{SSR(x_2, x_3|x_1)}{2}\div \dfrac{SSE(x_1,x_2, x_3)}{n-4}\\&=\dfrac{MSR(x_2, x_3|x_1)}{MSE(x_1,x_2, x_3)}.\end{align}

To conduct the test, we regress y = InfSize on \(x_{1}\) = Area and \(x_{2}\) and \(x_{3 }\)— in order (and with "Sequential sums of squares" selected under "Options"):

Source DF Seq SS Seq MS F- Value P-Value
Regression 3 0.95927 0.31976 16.43 0.000
Area 1 0.62492 0.63492 32.11 0.000
X2 1 0.3143 0.31453 16.16 0.001
X3 1 0.01981 0.01981 1.02 0.322
Error 28 0.54491 0.01946    
31 1.50418      

Inf = - 0.135 + 0.613 Area - 0.2435 X2 - 0.0657 X3

yielding SSR (\(x_{2}\) | \(x_{1}\)) = 0.31453, SSR (\(x_{3}\) | \(x_{1}\), \(x_{2}\)) = 0.01981, and MSE = 0.54491/28 = 0.01946. Therefore, the value of the partial F -statistic is:

\begin{align} F^*&=\dfrac{SSR(x_2, x_3|x_1)}{2}\div \dfrac{SSE(x_1,x_2, x_3)}{n-4}\\&=\dfrac{0.31453+0.01981}{2}\div\dfrac{0.54491}{28}\\&= \dfrac{0.33434}{2} \div 0.01946\\&=8.59,\end{align}

which is identical (within round-off error) to the general F-statistic above. The P -value is the probability — if the null hypothesis were true — that we would observe a partial F -statistic more extreme than 8.59. The following Minitab output:

F distribution with 2 DF in Numerator and 28 DF in denominator

x P ( X ≤ x )
8.59 0.998767

tells us that the probability of observing such an F -statistic that is smaller than 8.59 is 0.9988. Therefore, the probability of observing such an F -statistic that is larger than 8.59 is 1 - 0.9988 = 0.0012. The P -value is very small. There is sufficient evidence ( F = 8.59, P = 0.0012) to conclude that the type of cooling is significantly related to the extent of damage that occurs — after taking into account the size of the region at risk.

Summary of MLR Testing Section  

For the simple linear regression model, there is only one slope parameter about which one can perform hypothesis tests. For the multiple linear regression model, there are three different hypothesis tests for slopes that one could conduct. They are:

  • Hypothesis test for testing that all of the slope parameters are 0.
  • Hypothesis test for testing that a subset — more than one, but not all — of the slope parameters are 0.
  • Hypothesis test for testing that one slope parameter is 0.

We have learned how to perform each of the above three hypothesis tests. Along the way, we also took two detours — one to learn about the " general linear F-test " and one to learn about " sequential sums of squares. " As you now know, knowledge about both is necessary for performing the three hypothesis tests.

The F -statistic and associated p -value in the ANOVA table is used for testing whether all of the slope parameters are 0. In most applications, this p -value will be small enough to reject the null hypothesis and conclude that at least one predictor is useful in the model. For example, for the rabbit heart attacks study, the F -statistic is (0.95927/(4–1)) / (0.54491/(32–4)) = 16.43 with p -value 0.000.

To test whether a subset — more than one, but not all — of the slope parameters are 0, there are two equivalent ways to calculate the F-statistic:

  • Use the general linear F-test formula by fitting the full model to find SSE(F) and fitting the reduced model to find SSE(R) . Then the numerator of the F-statistic is (SSE(R) – SSE(F)) / ( \(df_{R}\) – \(df_{F}\)) .
  • Alternatively, use the partial F-test formula by fitting only the full model but making sure the relevant predictors are fitted last and "sequential sums of squares" have been selected. Then the numerator of the F-statistic is the sum of the relevant sequential sums of squares divided by the sum of the degrees of freedom for these sequential sums of squares. The denominator of the F -statistic is the mean squared error in the ANOVA table.

For example, for the rabbit heart attacks study, the general linear F-statistic is ((0.8793 – 0.54491) / (30 – 28)) / (0.54491 / 28) = 8.59 with p -value 0.0012. Alternatively, the partial F -statistic for testing the slope parameters for predictors \(x_{2}\) and \(x_{3}\) using sequential sums of squares is ((0.31453 + 0.01981) / 2) / (0.54491 / 28) = 8.59.

To test whether one slope parameter is 0, we can use an F -test as just described. Alternatively, we can use a t -test, which will have an identical p -value since in this case, the square of the t -statistic is equal to the F -statistic. For example, for the rabbit heart attacks study, the F -statistic for testing the slope parameter for the Area predictor is (0.63742/1) / (0.54491/(32–4)) = 32.75 with p -value 0.000. Alternatively, the t -statistic for testing the slope parameter for the Area predictor is 0.613 / 0.107 = 5.72 with p -value 0.000, and \(5.72^{2} = 32.72\).

Incidentally, you may be wondering why we can't just do a series of individual t-tests to test whether a subset of the slope parameters is 0. For example, for the rabbit heart attacks study, we could have done the following:

  • Fit the model of y = InfSize on \(x_{1}\) = Area and \(x_{2}\) and \(x_{3}\) and use an individual t-test for \(x_{3}\).
  • If the test results indicate that we can drop \(x_{3}\) then fit the model of y = InfSize on \(x_{1}\) = Area and \(x_{2}\) and use an individual t-test for \(x_{2}\).

The problem with this approach is we're using two individual t-tests instead of one F-test, which means our chance of drawing an incorrect conclusion in our testing procedure is higher. Every time we do a hypothesis test, we can draw an incorrect conclusion by:

  • rejecting a true null hypothesis, i.e., make a type I error by concluding the tested predictor(s) should be retained in the model when in truth it/they should be dropped; or
  • failing to reject a false null hypothesis, i.e., make a type II error by concluding the tested predictor(s) should be dropped from the model when in truth it/they should be retained.

Thus, in general, the fewer tests we perform the better. In this case, this means that wherever possible using one F-test in place of multiple individual t-tests is preferable.

Hypothesis tests for the slope parameters Section  

The problems in this section are designed to review the hypothesis tests for the slope parameters, as well as to give you some practice on models with a three-group qualitative variable (which we'll cover in more detail in Lesson 8). We consider tests for:

  • whether one slope parameter is 0 (for example, \(H_{0} \colon \beta_{1} = 0 \))
  • whether a subset (more than one but less than all) of the slope parameters are 0 (for example, \(H_{0} \colon \beta_{2} = \beta_{3} = 0 \) against the alternative \(H_{A} \colon \beta_{2} \ne 0 \) or \(\beta_{3} \ne 0 \) or both ≠ 0)
  • whether all of the slope parameters are 0 (for example, \(H_{0} \colon \beta_{1} = \beta_{2} = \beta_{3}\) = 0 against the alternative \(H_{A} \colon \) at least one of the \(\beta_{i}\) is not 0)

(Note the correct specification of the alternative hypotheses for the last two situations.)

Sugar beets study

A group of researchers was interested in studying the effects of three different growth regulators ( treat , denoted 1, 2, and 3) on the yield of sugar beets (y = yield , in pounds). They planned to plant the beets in 30 different plots and then randomly treat 10 plots with the first growth regulator, 10 plots with the second growth regulator, and 10 plots with the third growth regulator. One problem, though, is that the amount of available nitrogen in the 30 different plots varies naturally, thereby giving a potentially unfair advantage to plots with higher levels of available nitrogen. Therefore, the researchers also measured and recorded the available nitrogen (\(x_{1}\) = nit , in pounds/acre) in each plot. They are interested in comparing the mean yields of sugar beets subjected to the different growth regulators after taking into account the available nitrogen. The Sugar Beets dataset contains the data from the researcher's experiment.

Preliminary Work

The plot shows a similar positive linear trend within each treatment category, which suggests that it is reasonable to formulate a multiple regression model that would place three parallel lines through the data.

Because the qualitative variable treat distinguishes between the three treatment groups (1, 2, and 3), we need to create two indicator variables, \(x_{2}\) and \(x_{3}\), say, to fit a linear regression model to these data. The new indicator variables should be defined as follows:

treat \(x_2\) \(x_3\)
1 1 0
2 0 1
3 0 0

Use Minitab's Calc >> Make Indicator Variables command to create the new indicator variables in your worksheet

Minitab creates an indicator variable for each treatment group but we can only use two, for treatment groups 1 and 2 in this case (treatment group 3 is the reference level in this case).

Then, if we assume the trend in the data can be summarized by this regression model:

\(y_{i} = \beta_{0}\) + \(\beta_{1}\)\(x_{1}\) + \(\beta_{2}\)\(x_{2}\) + \(\beta_{3}\)\(x_{3}\) + \(\epsilon_{i}\)

where \(x_{1}\) = nit and \(x_{2}\) and \(x_{3}\) are defined as above, what is the mean response function for plots receiving treatment 3? for plots receiving treatment 1? for plots receiving treatment 2? Are the three regression lines that arise from our formulated model parallel? What does the parameter \(\beta_{2}\) quantify? And, what does the parameter \(\beta_{3}\) quantify?

The fitted equation from Minitab is Yield = 84.99 + 1.3088 Nit - 2.43 \(x_{2}\) - 2.35 \(x_{3}\), which means that the equations for each treatment group are:

  • Group 1: Yield = 84.99 + 1.3088 Nit - 2.43(1) = 82.56 + 1.3088 Nit
  • Group 2: Yield = 84.99 + 1.3088 Nit - 2.35(1) = 82.64 + 1.3088 Nit
  • Group 3: Yield = 84.99 + 1.3088 Nit

The three estimated regression lines are parallel since they have the same slope, 1.3088.

The regression parameter for \(x_{2}\) represents the difference between the estimated intercept for treatment 1 and the estimated intercept for reference treatment 3.

The regression parameter for \(x_{3}\) represents the difference between the estimated intercept for treatment 2 and the estimated intercept for reference treatment 3.

Testing whether all of the slope parameters are 0

\(H_0 \colon \beta_1 = \beta_2 = \beta_3 = 0\) against the alternative \(H_A \colon \) at least one of the \(\beta_i\) is not 0.

\(F=\dfrac{SSR(X_1,X_2,X_3)\div3}{SSE(X_1,X_2,X_3)\div(n-4)}=\dfrac{MSR(X_1,X_2,X_3)}{MSE(X_1,X_2,X_3)}\)

\(F = \dfrac{\frac{16039.5}{3}}{\frac{1078.0}{30-4}} = \dfrac{5346.5}{41.46} = 128.95\)

Since the p -value for this F -statistic is reported as 0.000, we reject \(H_{0}\) in favor of \(H_{A}\) and conclude that at least one of the slope parameters is not zero, i.e., the regression model containing at least one predictor is useful in predicting the size of sugar beet yield.

Tests for whether one slope parameter is 0

\(H_0 \colon \beta_1= 0\) against the alternative \(H_A \colon \beta_1 \ne 0\)

t -statistic = 19.60, p -value = 0.000, so we reject \(H_{0}\) in favor of \(H_{A}\) and conclude that the slope parameter for \(x_{1}\) = nit is not zero, i.e., sugar beet yield is significantly linearly related to the available nitrogen (controlling for treatment).

\(F=\dfrac{SSR(X_1|X_2,X_3)\div1}{SSE(X_1,X_2,X_3)\div(n-4)}=\dfrac{MSR(X_1|X_2,X_3)}{MSE(X_1,X_2,X_3)}\)

Use the Minitab output to calculate the value of this F statistic. Does the value you obtain equal \(t^{2}\), the square of the t -statistic as we might expect?

\(F-statistic= \dfrac{\frac{15934.5}{1}}{\frac{1078.0}{30-4}} = \dfrac{15934.5}{41.46} = 384.32\), which is the same as \(19.60^{2}\).

Because \(t^{2}\) will equal the partial F -statistic whenever you test for whether one slope parameter is 0, it makes sense to just use the t -statistic and P -value that Minitab displays as a default. But, note that we've just learned something new about the meaning of the t -test in the multiple regression setting. It tests for the ("marginal") significance of the \(x_{1}\) predictor after \(x_{2}\) and \(x_{3}\) have already been taken into account.

Tests for whether a subset of the slope parameters is 0

\(H_0 \colon \beta_2=\beta_3= 0\) against the alternative \(H_A \colon \beta_2 \ne 0\) or \(\beta_3 \ne 0\) or both \(\ne 0\).

\(F=\dfrac{SSR(X_2,X_3|X_1)\div2}{SSE(X_1,X_2,X_3)\div(n-4)}=\dfrac{MSR(X_2,X_3|X_1)}{MSE(X_1,X_2,X_3)}\)

\(F = \dfrac{\frac{10.4+27.5}{2}}{\frac{1078.0}{30-4}} = \dfrac{18.95}{41.46} = 0.46\).

F distribution with 2 DF in Numerator and 26 DF in denominator

x P ( X ≤ x )
0.46 0.363677

p-value \(= 1-0.363677 = 0.636\), so we fail to reject \(H_{0}\) in favor of \(H_{A}\) and conclude that we cannot rule out \(\beta_2 = \beta_3 = 0\), i.e., there is no significant difference in the mean yields of sugar beets subjected to the different growth regulators after taking into account the available nitrogen.

Note that the sequential mean square due to regression, MSR(\(X_{2}\),\(X_{3}\)|\(X_{1}\)), is obtained by dividing the sequential sum of square by its degrees of freedom (2, in this case, since two additional predictors \(X_{2}\) and \(X_{3}\) are considered). Use the Minitab output to calculate the value of this F statistic, and use Minitab to get the associated P -value. Answer the researcher's question at the \(\alpha= 0.05\) level.

Logo for Open Library Publishing Platform

Want to create or adapt books like this? Learn more about how Pressbooks supports open publishing practices.

13.6 Testing the Regression Coefficients

Learning objectives.

  • Conduct and interpret a hypothesis test on individual regression coefficients.

Previously, we learned that the population model for the multiple regression equation is

[latex]\begin{eqnarray*} y & = & \beta_0+\beta_1x_1+\beta_2x_2+\cdots+\beta_kx_k +\epsilon \end{eqnarray*}[/latex]

where [latex]x_1,x_2,\ldots,x_k[/latex] are the independent variables, [latex]\beta_0,\beta_1,\ldots,\beta_k[/latex] are the population parameters of the regression coefficients, and [latex]\epsilon[/latex] is the error variable.  In multiple regression, we estimate each population regression coefficient [latex]\beta_i[/latex] with the sample regression coefficient [latex]b_i[/latex].

In the previous section, we learned how to conduct an overall model test to determine if the regression model is valid.  If the outcome of the overall model test is that the model is valid, then at least one of the independent variables is related to the dependent variable—in other words, at least one of the regression coefficients [latex]\beta_i[/latex] is not zero.  However, the overall model test does not tell us which independent variables are related to the dependent variable.  To determine which independent variables are related to the dependent variable, we must test each of the regression coefficients.

Testing the Regression Coefficients

For an individual regression coefficient, we want to test if there is a relationship between the dependent variable [latex]y[/latex] and the independent variable [latex]x_i[/latex].

  • No Relationship .  There is no relationship between the dependent variable [latex]y[/latex] and the independent variable [latex]x_i[/latex].  In this case, the regression coefficient [latex]\beta_i[/latex] is zero.  This is the claim for the null hypothesis in an individual regression coefficient test:  [latex]H_0: \beta_i=0[/latex].
  • Relationship.  There is a relationship between the dependent variable [latex]y[/latex] and the independent variable [latex]x_i[/latex].  In this case, the regression coefficients [latex]\beta_i[/latex] is not zero.  This is the claim for the alternative hypothesis in an individual regression coefficient test:  [latex]H_a: \beta_i \neq 0[/latex].  We are not interested if the regression coefficient [latex]\beta_i[/latex] is positive or negative, only that it is not zero.  We only need to find out if the regression coefficient is not zero to demonstrate that there is a relationship between the dependent variable and the independent variable. This makes the test on a regression coefficient a two-tailed test.

In order to conduct a hypothesis test on an individual regression coefficient [latex]\beta_i[/latex], we need to use the distribution of the sample regression coefficient [latex]b_i[/latex]:

  • The mean of the distribution of the sample regression coefficient is the population regression coefficient [latex]\beta_i[/latex].
  • The standard deviation of the distribution of the sample regression coefficient is [latex]\sigma_{b_i}[/latex].  Because we do not know the population standard deviation we must estimate [latex]\sigma_{b_i}[/latex] with the sample standard deviation [latex]s_{b_i}[/latex].
  • The distribution of the sample regression coefficient follows a normal distribution.

Steps to Conduct a Hypothesis Test on a Regression Coefficient

[latex]\begin{eqnarray*} H_0: &  &  \beta_i=0 \\ \\ \end{eqnarray*}[/latex]

[latex]\begin{eqnarray*} H_a: &  & \beta_i \neq 0 \\ \\ \end{eqnarray*}[/latex]

  • Collect the sample information for the test and identify the significance level [latex]\alpha[/latex].

[latex]\begin{eqnarray*}t & = & \frac{b_i-\beta_i}{s_{b_i}} \\ \\ df &  = & n-k-1 \\  \\ \end{eqnarray*}[/latex]

  • The results of the sample data are significant.  There is sufficient evidence to conclude that the null hypothesis [latex]H_0[/latex] is an incorrect belief and that the alternative hypothesis [latex]H_a[/latex] is most likely correct.
  • The results of the sample data are not significant.  There is not sufficient evidence to conclude that the alternative hypothesis [latex]H_a[/latex] may be correct.
  • Write down a concluding sentence specific to the context of the question.

The required [latex]t[/latex]-score and p -value for the test can be found on the regression summary table, which we learned how to generate in Excel in a previous section.

The human resources department at a large company wants to develop a model to predict an employee’s job satisfaction from the number of hours of unpaid work per week the employee does, the employee’s age, and the employee’s income.  A sample of 25 employees at the company is taken and the data is recorded in the table below.  The employee’s income is recorded in $1000s and the job satisfaction score is out of 10, with higher values indicating greater job satisfaction.

4 3 23 60
5 8 32 114
2 9 28 45
6 4 60 187
7 3 62 175
8 1 43 125
7 6 60 93
3 3 37 57
5 2 24 47
5 5 64 128
7 2 28 66
8 1 66 146
5 7 35 89
2 5 37 56
4 0 59 65
6 2 32 95
5 6 76 82
7 5 25 90
9 0 55 137
8 3 34 91
7 5 54 184
9 1 57 60
7 0 68 39
10 2 66 187
5 0 50 49

Previously, we found the multiple regression equation to predict the job satisfaction score from the other variables:

[latex]\begin{eqnarray*} \hat{y} & = & 4.7993-0.3818x_1+0.0046x_2+0.0233x_3 \\ \\ \hat{y} & = & \mbox{predicted job satisfaction score} \\ x_1 & = & \mbox{hours of unpaid work per week} \\ x_2 & = & \mbox{age} \\ x_3 & = & \mbox{income (\$1000s)}\end{eqnarray*}[/latex]

At the 5% significance level, test the relationship between the dependent variable “job satisfaction” and the independent variable “hours of unpaid work per week”.

Hypotheses:

[latex]\begin{eqnarray*} H_0: & & \beta_1=0 \\   H_a: & & \beta_1 \neq 0 \end{eqnarray*}[/latex]

The regression summary table generated by Excel is shown below:

Multiple R 0.711779225
R Square 0.506629665
Adjusted R Square 0.436148189
Standard Error 1.585212784
Observations 25
Regression 3 54.189109 18.06303633 7.18812504 0.001683189
Residual 21 52.770891 2.512899571
Total 24 106.96
Intercept 4.799258185 1.197185164 4.008785216 0.00063622 2.309575344 7.288941027
Hours of Unpaid Work per Week -0.38184722 0.130750479 -2.9204269 0.008177146 -0.65375772 -0.10993671
Age 0.004555815 0.022855709 0.199329423 0.843922453 -0.04297523 0.052086864
Income ($1000s) 0.023250418 0.007610353 3.055103771 0.006012895 0.007423823 0.039077013

The  p -value for the test on the hours of unpaid work per week regression coefficient is in the bottom part of the table under the P-value column of the Hours of Unpaid Work per Week row .  So the  p -value=[latex]0.0082[/latex].

Conclusion:  

Because p -value[latex]=0.0082 \lt 0.05=\alpha[/latex], we reject the null hypothesis in favour of the alternative hypothesis.  At the 5% significance level there is enough evidence to suggest that there is a relationship between the dependent variable “job satisfaction” and the independent variable “hours of unpaid work per week.”

  • The null hypothesis [latex]\beta_1=0[/latex] is the claim that the regression coefficient for the independent variable [latex]x_1[/latex] is zero.  That is, the null hypothesis is the claim that there is no relationship between the dependent variable and the independent variable “hours of unpaid work per week.”
  • The alternative hypothesis is the claim that the regression coefficient for the independent variable [latex]x_1[/latex] is not zero.  The alternative hypothesis is the claim that there is a relationship between the dependent variable and the independent variable “hours of unpaid work per week.”
  • When conducting a test on a regression coefficient, make sure to use the correct subscript on [latex]\beta[/latex] to correspond to how the independent variables were defined in the regression model and which independent variable is being tested.  Here the subscript on [latex]\beta[/latex] is 1 because the “hours of unpaid work per week” is defined as [latex]x_1[/latex] in the regression model.
  • The p -value for the tests on the regression coefficients are located in the bottom part of the table under the P-value column heading in the corresponding independent variable row. 
  • Because the alternative hypothesis is a [latex]\neq[/latex], the p -value is the sum of the area in the tails of the [latex]t[/latex]-distribution.  This is the value calculated out by Excel in the regression summary table.
  • The p -value of 0.0082 is a small probability compared to the significance level, and so is unlikely to happen assuming the null hypothesis is true.  This suggests that the assumption that the null hypothesis is true is most likely incorrect, and so the conclusion of the test is to reject the null hypothesis in favour of the alternative hypothesis.  In other words, the regression coefficient [latex]\beta_1[/latex] is not zero, and so there is a relationship between the dependent variable “job satisfaction” and the independent variable “hours of unpaid work per week.”  This means that the independent variable “hours of unpaid work per week” is useful in predicting the dependent variable.

At the 5% significance level, test the relationship between the dependent variable “job satisfaction” and the independent variable “age”.

[latex]\begin{eqnarray*} H_0: & & \beta_2=0 \\   H_a: & & \beta_2 \neq 0 \end{eqnarray*}[/latex]

The  p -value for the test on the age regression coefficient is in the bottom part of the table under the P-value column of the Age row .  So the  p -value=[latex]0.8439[/latex].

Because p -value[latex]=0.8439 \gt 0.05=\alpha[/latex], we do not reject the null hypothesis.  At the 5% significance level there is not enough evidence to suggest that there is a relationship between the dependent variable “job satisfaction” and the independent variable “age.”

  • The null hypothesis [latex]\beta_2=0[/latex] is the claim that the regression coefficient for the independent variable [latex]x_2[/latex] is zero.  That is, the null hypothesis is the claim that there is no relationship between the dependent variable and the independent variable “age.”
  • The alternative hypothesis is the claim that the regression coefficient for the independent variable [latex]x_2[/latex] is not zero.  The alternative hypothesis is the claim that there is a relationship between the dependent variable and the independent variable “age.”
  • When conducting a test on a regression coefficient, make sure to use the correct subscript on [latex]\beta[/latex] to correspond to how the independent variables were defined in the regression model and which independent variable is being tested.  Here the subscript on [latex]\beta[/latex] is 2 because “age” is defined as [latex]x_2[/latex] in the regression model.
  • The p -value of 0.8439 is a large probability compared to the significance level, and so is likely to happen assuming the null hypothesis is true.  This suggests that the assumption that the null hypothesis is true is most likely correct, and so the conclusion of the test is to not reject the null hypothesis.  In other words, the regression coefficient [latex]\beta_2[/latex] is zero, and so there is no relationship between the dependent variable “job satisfaction” and the independent variable “age.”  This means that the independent variable “age” is not particularly useful in predicting the dependent variable.

At the 5% significance level, test the relationship between the dependent variable “job satisfaction” and the independent variable “income”.

[latex]\begin{eqnarray*} H_0: & & \beta_3=0 \\   H_a: & & \beta_3 \neq 0 \end{eqnarray*}[/latex]

The  p -value for the test on the income regression coefficient is in the bottom part of the table under the P-value column of the Income row .  So the  p -value=[latex]0.0060[/latex].

Because p -value[latex]=0.0060 \lt 0.05=\alpha[/latex], we reject the null hypothesis in favour of the alternative hypothesis.  At the 5% significance level there is enough evidence to suggest that there is a relationship between the dependent variable “job satisfaction” and the independent variable “income.”

  • The null hypothesis [latex]\beta_3=0[/latex] is the claim that the regression coefficient for the independent variable [latex]x_3[/latex] is zero.  That is, the null hypothesis is the claim that there is no relationship between the dependent variable and the independent variable “income.”
  • The alternative hypothesis is the claim that the regression coefficient for the independent variable [latex]x_3[/latex] is not zero.  The alternative hypothesis is the claim that there is a relationship between the dependent variable and the independent variable “income.”
  • When conducting a test on a regression coefficient, make sure to use the correct subscript on [latex]\beta[/latex] to correspond to how the independent variables were defined in the regression model and which independent variable is being tested.  Here the subscript on [latex]\beta[/latex] is 3 because “income” is defined as [latex]x_3[/latex] in the regression model.
  • The p -value of 0.0060 is a small probability compared to the significance level, and so is unlikely to happen assuming the null hypothesis is true.  This suggests that the assumption that the null hypothesis is true is most likely incorrect, and so the conclusion of the test is to reject the null hypothesis in favour of the alternative hypothesis.  In other words, the regression coefficient [latex]\beta_3[/latex] is not zero, and so there is a relationship between the dependent variable “job satisfaction” and the independent variable “income.”  This means that the independent variable “income” is useful in predicting the dependent variable.

Concept Review

The test on a regression coefficient determines if there is a relationship between the dependent variable and the corresponding independent variable.  The p -value for the test is the sum of the area in tails of the [latex]t[/latex]-distribution.  The p -value can be found on the regression summary table generated by Excel.

The hypothesis test for a regression coefficient is a well established process:

  • Write down the null and alternative hypotheses in terms of the regression coefficient being tested.  The null hypothesis is the claim that there is no relationship between the dependent variable and independent variable.  The alternative hypothesis is the claim that there is a relationship between the dependent variable and independent variable.
  • Collect the sample information for the test and identify the significance level.
  • The p -value is the sum of the area in the tails of the [latex]t[/latex]-distribution.  Use the regression summary table generated by Excel to find the p -value.
  • Compare the  p -value to the significance level and state the outcome of the test.

Introduction to Statistics Copyright © 2022 by Valerie Watts is licensed under a Creative Commons Attribution-NonCommercial-ShareAlike 4.0 International License , except where otherwise noted.

IMAGES

  1. Multiple Regression Assumptions & Diagnostics

    alternative hypothesis for multiple regression

  2. PPT

    alternative hypothesis for multiple regression

  3. SOLVED: Question 1 (3 points) Consider the following multiple

    alternative hypothesis for multiple regression

  4. Confidence Intervals in Multiple Regression

    alternative hypothesis for multiple regression

  5. Hypothesis Tests in Multiple Linear Regression, Part 1

    alternative hypothesis for multiple regression

  6. PPT

    alternative hypothesis for multiple regression

VIDEO

  1. Multiple regression, hypothesis testing, model deployment

  2. Regression and test of hypothesis

  3. Statistics and probability

  4. Statistics and probability- Test on an Individual Coefficient in Multiple Regression

  5. Hypothesis testing: Linear & Multiple Regression in R

  6. Multiple Categorical Predictors/One-Way ANOVA

COMMENTS

  1. Null and Alternative hypothesis for multiple linear regression

    I have 1 dependent variable and 3 independent variables. I run multiple regression, and find that the p value for one of the independent variables is higher than 0.05 (95% is my confidence level).

  2. Understanding the Null Hypothesis for Linear Regression

    xi: The value of the predictor variable xi. Multiple linear regression uses the following null and alternative hypotheses: H0: β1 = β2 = … = βk = 0. HA: β1 = β2 = … = βk ≠ 0. The null hypothesis states that all coefficients in the model are equal to zero. In other words, none of the predictor variables have a statistically ...

  3. PDF Lecture 5 Hypothesis Testing in Multiple Linear Regression

    Hypothesis Testing in Multiple Linear Regression BIOST 515 January 20, 2004. 1 Types of tests • Overall test • Test for addition of a single variable • Test for addition of a group of variables. 2 ... As in simple linear regression, under the null hypothesis t 0 =

  4. PDF Hypothesis Testing in the Multiple regression model

    Testing that individual coefficients take a specific value such as zero or some other value is done in exactly the same way as with the simple two variable regression model. Now suppose we wish to test that a number of coefficients or combinations of coefficients take some particular value. In this case we will use the so called "F-test".

  5. Multiple Linear Regression

    The formula for a multiple linear regression is: = the predicted value of the dependent variable. = the y-intercept (value of y when all other parameters are set to 0) = the regression coefficient () of the first independent variable () (a.k.a. the effect that increasing the value of the independent variable has on the predicted y value ...

  6. Writing hypothesis for linear multiple regression models

    2. I struggle writing hypothesis because I get very much confused by reference groups in the context of regression models. For my example I'm using the mtcars dataset. The predictors are wt (weight), cyl (number of cylinders), and gear (number of gears), and the outcome variable is mpg (miles per gallon). Say all your friends think you should ...

  7. Multiple Linear Regression

    Multiple Linear Regression: It's a form of linear regression that is used when there are two or more predictors. We will see how multiple input variables together influence the output variable, while also learning how the calculations differ from that of Simple LR model. ... Alternative Hypothesis. The hypothesis test is performed by using F ...

  8. Null Hypothesis for Multiple Regression

    In multiple regression analysis, a null hypothesis is a crucial concept that plays a central role in statistical inference and hypothesis testing. A null hypothesis, denoted by H0, is a statement that proposes no significant relationship between the independent variables and the dependent variable. In other words, the null hypothesis suggests ...

  9. Lesson 5: Multiple Linear Regression (MLR) Model & Evaluation

    a hypothesis test for testing that a subset — more than one, but not all — of the slope parameters are 0. In this lesson, we also learn how to perform each of the above three hypothesis tests. Key Learning Goals for this Lesson: Be able to interpret the coefficients of a multiple regression model. Understand what the scope of the model is ...

  10. 13.5 Testing the Significance of the Overall Model

    The p-value for the overall model test is in the middle part of the table under the ANOVA heading in the Significance F column of the Regression row. So the p-value=[latex]0.0017[/latex]. Conclusion: Because p-value[latex]=0.0017 \lt 0.05=\alpha[/latex], we reject the null hypothesis in favour of the alternative hypothesis. At the 5% ...

  11. Multiple Linear Regression

    For any of the variables x j included in a multiple regression model, the null hypothesis states that the coefficient j is equal to 0. The alternative hypothesis may be one-sided or two-sided, stating that j is either less than 0, greater than 0, or simply not equal to 0.

  12. Multiple linear regression -- Advanced Statistics using R

    The general purpose of multiple regression (the term was first used by Pearson, 1908), as a generalization of simple linear regression, is to learn about how several independent variables or predictors (IVs) together predict a dependent variable (DV). ... For this hypothesis testing, the null and alternative hypothesis are \[H_{0}:\beta_{1 ...

  13. PDF Multiple Hypothesis Testing: The F-test

    an Moore and Alison Post.2 The F -testWe have seen our t-statistic follows a t distribution wi. h a "degrees of freedom" parameter. This fact has been useful for hypothesis testing, both of samp. e means and of regression coefficients. We are able to test, say, the hypothesis that some variable. as no effect on the de.

  14. PDF Multiple Regression Analysis: Inference

    The upper tail probability can be calculated in STATA for a t distribution with "n" degrees of freedom using the function "ttail". Its syntax is: ttail(n,t), where "t" is the cut-off for the upper tail. You can also look up the value of "t" for which "p" is the upper tail area using "invttail".

  15. 5.3

    A population model for a multiple linear regression model that relates a y -variable to p -1 x -variables is written as. y i = β 0 + β 1 x i, 1 + β 2 x i, 2 + … + β p − 1 x i, p − 1 + ϵ i. We assume that the ϵ i have a normal distribution with mean 0 and constant variance σ 2. These are the same assumptions that we used in simple ...

  16. PDF Multiple Regression in SPSS STAT 314

    Test the alternative hypothesis that the mean profit margin with 3.5 net revenues and 6500 branches is greater than 0.70. Test at the 5% significance level. ... assumptions for multiple regression analysis are met by the variables in questions. The residuals [res] and standardized values [sre] (as well as the predicted values [pre], the ...

  17. Multiple Linear Regression in R: Tutorial With Examples

    Sum(Xi) represents the sum expression in the multiple linear regression equation. our_data is the churn_data. You can learn more from our Intermediate Regression in R course. An alternative to using R is the Intermediate Regression with statsmodels in Python. Both help you learn linear and logistic regression with multiple explanatory variables.

  18. 8.7: Overall F-test in multiple linear regression

    This test is called the overall F-test in MLR and is very similar to the F F -test in a reference-coded One-Way ANOVA model. It tests the null hypothesis that involves setting every coefficient except the y y -intercept to 0 (so all the slope coefficients equal 0). We saw this reduced model in the One-Way material when we considered setting all ...

  19. 6.4

    For the simple linear regression model, there is only one slope parameter about which one can perform hypothesis tests. For the multiple linear regression model, there are three different hypothesis tests for slopes that one could conduct. They are: Hypothesis test for testing that all of the slope parameters are 0.

  20. 13.6 Testing the Regression Coefficients

    The alternative hypothesis is the claim that the regression coefficient for the independent variable [latex]x_1[/latex] is not zero. The alternative hypothesis is the claim that there is a relationship between the dependent variable and the independent variable "hours of unpaid work per week."

  21. PDF 4 Hypothesis testing in the multiple regression model

    4.1.1 Formulation of the null hypothesis and the alternative hypothesis 2 4.1.2 Test statistic 2 ... Before testing hypotheses in the multiple regression model, we are going to offer

  22. Multiple regression

    Use multiple regression when you have three or more measurement variables. One of the measurement variables is the dependent ( Y) variable. The rest of the variables are the independent ( X) variables; you think they may have an effect on the dependent variable. The purpose of a multiple regression is to find an equation that best predicts the ...

  23. 12.2.1: Hypothesis Test for Linear Regression

    The two test statistic formulas are algebraically equal; however, the formulas are different and we use a different parameter in the hypotheses. The formula for the t-test statistic is t = b1 (MSE SSxx)√ t = b 1 ( M S E S S x x) Use the t-distribution with degrees of freedom equal to n − p − 1 n − p − 1.